IMMUNE DISORDERS /

Pataasin ang iyong marka sa homework at exams ngayon gamit ang Quizwiz!

Preventing Transfusion reactions

-careful selection of blood donors, followed by careful typing and cross-matching of blood from donor to recipient. -Proper storage of blood and adherence to the administration protocol are critical. --Blood and blood components must be refrigerated at specific temperatures until a half hour before administration. -Blood must be administered within 4 hours of removal from refrigeration, and blood components within 6 hours. -Donor and recipient numbers are specific and must be checked thoroughly, and the patient must be identified with an armband. -Administer all blood and blood products through microaggregate filters. Monitor for adverse effects.

Skin testing

. Description a. The administration of an allergen to the surface of the skin or into the dermis b. Administered by patch, scratch, or intradermal techniques 2. Preprocedure interventions a. Discontinue systemic corticosteroids or antihistamine therapy 5 days before the test, as prescribed. b. Ensure informed consent has been obtained. 3. Postprocedure interventions a. Record the site, date, and time of the test. b. Record the date and the time for follow-up site reading. c. Have client remain in waiting room or office for at least 30 minutes after the injection to monitor for adverse effects. d. Inspect the site for erythema, papules, vesicles, edema, and wheal. e. Measure flare along with the wheal, and document size and other findings; a wheal of 0.5 cm in diameter or greater is considered positive. f. Provide the client with a list of potential allergens, if identified. Have resus

A 34-year-old patient comes to the clinic requesting HIV testing. He is a gay man with two significant others in his lifetime. His partner recently received a diagnosis of HIV infection. The patient asks the nurse how long it will take before the infection could show up in him. What would be the nurse's best response? 1. "Antibodies may be detected in the blood within 1 to 12 weeks of exposure." 2. "It takes at least a year to know if you will be infected with HIV." 3. "You'll have to ask your doctor the next time you go in for an appointment." 4. "We don't know how long it will take for a person to seroconvert to being HIV positive."

1. "Antibodies may be detected in the blood within 1 to 12 weeks of exposure."

A 72-year-old patient is admitted to the hospital with a diagnosis of immunodeficiency disease. What is the primary nursing goal? 1. Reduce the risk of the patient developing an infection. 2. Encourage the patient to provide self-care. 3. Plan nutritious meals to provide adequate intake. 4. Encourage the patient to interact with other patients.

1. Reduce the risk of the patient developing an infection.

Which illnesses are believed to be caused by an autoimmune disorder? (Select all that apply.) 1. Rheumatoid arthritis 2. Lung cancer 3. Systemic lupus erythematosus 4. Guillain-Barré syndrome 5. Cardiovascular disease

1. Rheumatoid arthritis 2. Lung cancer 3. Systemic lupus erythematosus

earliest case of HIV

1959 It was identified in the Democratic Republic of Congo through different methods of molecular clock analysis. It also has been estimated that HIV began to radiate from its source in approximately the 1930s.

The nurse advises a friend who asks the nurse to administer his allergy injections that: 1. it is illegal for nurses to administer injections outside of a medical setting. 2. he is qualified to do it if the friend has epinephrine in an injectable syringe provided with his extract. 3. avoiding the allergens is a more effective way of controlling allergies and allergy shots are not usually effective. 4. immunotherapy should be administered only in a setting where emergency equipment and drugs are available.

4. immunotherapy should be administered only in a setting where emergency equipment and drugs are available.

Genetic Control of Immunity

A genetic link exists to well-developed immune systems and poorly developed or compromised immune systems.

Complement System

A group of about 30 blood proteins that may amplify the inflammatory response, enhance phagocytosis, or directly lyse extracellular pathogens. The complement system functions in a "step-by-step" series much like the clotting mechanism.

Western blot

A laboratory blood test to detect the presence of antibodies to a specific antigen; used in diagnosing HIV.

What is the most accurate statement regarding latex allergies?

A person with a latex allergy should be taught to wear a medical alert bracelet and carry an epinephrine pen.

immunization

A process by which resistance to an infectious disease is induced or increased.

immunotherapy

A special treatment of allergic responses; involves the administration of increasingly larger doses of the offending allergens to gradually develop immunity.

antigen

A substance recognized by the body as foreign that can trigger an immune response.

allergen

A substance that can produce a hypersensitive reaction in the body but that is not necessarily inherently harmful.

CD4 + lymphocyte

A type of white blood cell; a protein on the surface of cells that normally helps the body's immune system combat disease.

Medications for Hypersensitivity Reactions- epinephrine (Epinephrine chloride, EpiPen)

ACTION- Alphaadrenergic agonist and betaadrenergic agonist. SIDE EFFECTS- Nervousness, tremor, headache, hypertension, tachycardia, ventricular fibrillation, stroke. NURSING IMPLICATIONS- Do not use with monoamine oxidase inhibitor; use cautiously in patients with hyperthyroidism, hypertension, diabetes, and heart disease.

Medications for Hypersensitivity Reactions-fexofenadine (Allegra)

ACTION- Nonsedating antihistamine. SIDE EFFECTS- Headache, drowsiness, blurred vision, hypotension, bradycardia, tachycardia, dysrhythmias (rare), urinary retention. NURSING IMPLICATIONS- Same as for loratadine.

Medications for Hypersensitivity Reactions-loratadine (Claritin)

ACTIONS- Nonsedating antihistamine. SIDE EFFECTS- Dry mouth, headache, red and irritated eyes. NURSING IMPLICATIONS- Store in tight container at room temperature. Teach patient and family to avoid driving or other hazardous activities if drowsiness occurs.

Types of Acquired Immunity

ACTIVE Natural immunity: Results from exposure to the infectious agent Artificial immunity: Results from immunization or vaccination with an antigen

What statement is most accurate concerning AIDS?

AIDS is used to describe the end-stage, or terminal phase, of the HIV infection.

immunosuppressive

Administration of agents that significantly interfere with the ability of the immune system to respond to antigenic stimulation by inhibiting cellular and humoral immunity.

viral load

Amount of measurable HIV virions in the blood

Types of White Blood Cells and Their Involvement in HIV Disease- BASOPHILS, MAST CELLS

Both: involved in acute inflammation Mast cells: breakdown releases histamine and other factors. ROLE IN HIV DISEASE In HIV infection, these cells may inhibit leukocyte migration.

What is the highest priority patient problem for the patient experiencing anaphylaxis?

Breathing pattern, ineffective

What are considered methods of improving safety for blood transfusions? (Select all that apply.)

Careful selection of blood donors Use of microaggregate filters for blood administration Close monitoring of the patient receiving a blood transfusion Carefully checking donor and recipient numbers before administering blood

Combination drug therapy is now the standard of care.

Combination drug therapy is now the standard of care.

Types of White Blood Cells and Their Involvement in HIV Disease- MONOCYTES, MACROPHAGES

Constitute about 3%- 7% of all WBCs Involved in the inflammatory response Capable of processing antigens for presentation to T cells Have CD4 + receptors Macrophages: distributed throughout tissue and capable of phagocytosis. ROLE IN HIV DISEASE. Monocytes and macrophages serve as a reservoir for HIV. When activated by stimulation with interferon (inflammatory response), they produce neopterin. Neopterin levels are increased in HIV disease.

What is the most common type of a secondary immunodeficiency disorder?

Drug induced

Acute Intervention

Early intervention after detection of an HIV infection can promote health and limit or delay disability. Because the course of HIV infection is extremely variable, assessment is of primary importance. Nursing interventions are tailored to any patient needs noted during assessment. The nursing assessment of HIV disease should focus on the early detection of constitutional symptoms, opportunistic diseases, and psychosocial problems

What is the most effective treatment for a hypersensitivity disorder?

Environmental control

adherence

Following a prescribed regimen of therapy or treatment for disease.

Counseling Before and After HIV Antibody Testing

General Guidelines • People who are being tested for HIV are commonly fearful of the test results; therefore carry out the following steps: • Establish rapport with the patient. • Assess the patient's ability to understand counseling. • Determine the patient's access to support systems. • Explain the following benefits of testing: • Testing provides an opportunity for education that can decrease the risk of new infections. • Infected patients can be referred for early intervention and support programs. • Discuss the following negative aspects of testing: • Breaches of confidentiality have led to discrimination. • A positive test result affects all aspects of the patient's life (personal, social, economic) and can raise difficult emotions (anger, anxiety, guilt, thoughts of suicide). Pretest Counseling • Determine the patient's risk factors and when the last exposure risk occurred. Individualize counseling according to these parameters. • Provide education to decrease future risk of exposure. • Provide education that will help the patient protect sexual and drug-sharing partners. • Discuss problems related to the delay between infection and an accurate test result: • Testing must be repeated at intervals for 6 months after each possible exposure. • The patient needs to abstain from further high-risk behaviors during that interval. • The patient needs to protect sexual and drug-sharing partners during that interval. • Discuss the possibility of false-negative test results, which are most likely to occur during the period between infection and seroconversion. • Provide information regarding the confidentiality of results. • Explain that a positive test result shows HIV infection, not AIDS. • Explain that the test result does not establish immunity. • Assess the patient's support systems; provide telephone numbers and resources as needed. • Discuss the patient's personally anticipated responses to test results (positive and negative). • Outline assistance that will be offered if the test result is positive. Posttest Counseling • If the test result is negative, reinforce pretest counseling and prevention education. Remind the patient that test must be repeated at intervals for 6 months after the most recent exposure risk. • Explain the need to report positive test results to the state health department and CDC. • Discuss the importance of partner notification. • If the test result is positive, understand that the patient may be in shock and not hear what is said after receiving that news. • Provide resources for medical and emotional support, and help the patient get immediate assistance. • Evaluate the patient's suicide risk, and follow up as needed. • Determine need to test other people who have had high-risk contact with the patient. • Discuss retesting to verify results. This tactic provides hope for the patient, but, of more importance, it keeps the patient in touch with health care professionals. While waiting for the second test result, the patient has time to think about and adjust to the possibility of being infected with HIV. • Encourage optimism: • Remind the patient that treatments are available. • Review health habits that can improve the immune system. • Arrange for patients to speak to HIV-infected people who are willing to share and assist patients with new diagnoses during the transition period. • Reinforce that an HIV-positive test result means the patient is infected but does not necessarily mean that the patient has AIDS. AIDS, Acquired immunodeficiency syndrome; HIV, human immunodeficiency virus.

virulent

Having the power to produce disease; of or pertaining to a very pathogenic or rapidly progressive condition.

Common Allergens Causing Anaphylaxis-VENOMS

Honeybees Wasps Hornets

Review of Mechanisms of the Immune Response

Humoral immunity responds to antigens such as bacteria and foreign tissue. Humoral immunity is the result of the development and continuing presence of circulating antibodies in the plasma. Humoral immunity consists of antibody-mediated immunity. Humoral means body fluid, and antibodies are proteins found in plasma. Therefore the term humoral immunity Cellular immunity is the primary defense against intracellular organisms, including viruses and some bacteria (e.g., mycobacteria).

Review of Mechanisms of the Immune Response

Hypersensitivity reaction is an inappropriate and excessive response of the immune system to a sensitizing antigen. In hypersensitivity reactions, the antigen is an allergen. Humoral reactions, such as anaphylactic hypersensitivity, are immediate. Cellular reactions are delayed reactions. Examples of delayed reactions include contact dermatitis, tissue transplant rejection, and sarcoidosis (Ignatavicius and Workman, 2016).

Latex allergy- Nursing Interventions

Identification of patients and health care workers sensitive to latex is crucial in preventing adverse reactions. Collect a thorough health history and history of any allergies, especially for patients with any complaints of latex contact symptoms.

adaptive, acquired immune response assists in the battle

If the components of innate or natural immunity fail to prevent invasion or to destroy a foreign pathogen. bodies 2nd line of defense against disease.

autoimmune

Immune response (autoantibodies or cellular immune response) to one's own tissues.

attenuated

In obstetrics, the relationship of fetal body parts to one another.

Proper Terms Related to HIV and AIDS

MISLEADING- high-risk groups infection with AIDs AIDs test AIDs positive AIDs victim or patient AIDs carrier. MORE ACCURATE PHRASES high-risk behavior high-risk infection HIV antibody test HIV positive Person living with HIV or AIDs HIV-infected person

Medications for Hypersensitivity Reactions

Medications are used to treat and alleviate signs and symptoms.

seronegative

Negative result on serologic examination. The state of lacking HIV antibodies; confirmed by blood tests.

lymphokines

One of the chemical factors produced and released by T lymphocytes that attract macrophages to the site of infection or inflammation and prepare them for attack.

humoral immunity

One of the two forms of immunity that respond to antigens such as bacteria and foreign tissue. It is mediated by B cells.

Types of Acquired Immunity

PASSIVE Natural immunity: Results from the passage of maternal antibodies from mother to child Artificial immunity: Results from the injection of antibodies from other sources

Opportunistic Diseases Associated With HIV

Probably the most difficult aspect of the medical management of HIV is dealing with the many opportunistic diseases that develop as the immune system degenerates. Although it is usually impossible to totally eradicate opportunistic diseases, the use of ART and prophylactic interventions can control their emergence or progression.

Types of White Blood Cells and Their Involvement in HIV Disease-B CELLS

Produce antibodies specific to an antigen Capable of being stimulated by T helper cells. ROLE IN HIV B cells are involved in the humoral response to HIV infection and produce a variety of antibodies against HIV. They are present throughout the course of HIV disease.

adaptive immunity

Protection that provides a specific reaction to each invading antigen and has the unique ability to remember the antigen that caused the attack.

Kaposi's sarcoma KS

Rare cancer of the skin or mucous membranes; characterized by blue, red, or purple raised lesions and seen mainly in middle-aged Mediterranean men and those with HIV disease.

proliferate

Reproduction or multiplication of similar forms.

Regulate its action

Self-regulation allows the immune system to monitor itself by "turning on" when an antigen invades and "turning off" when the invasion has been eradicated. Regulation prevents the destruction of healthy or host tissue. The inability to regulate could result in a chronic inflammation and damage to the host tissue.

Innate (Natural) and Adaptive (Acquired) Immunity

Soluble factors CHARACTERISTICS INNATE (NATURAL) Chemical defense: Lysozyme, complement, acutephase proteins, interferon ADAPTIVE (ACQUIRED) Antibodies, lymphokines

autologous

Something that has its origin within an individual, especially a factor present in tissues or fluids.

Innate (Natural) and Adaptive (Acquired) Immunity

Specificity CHARACTERISTICS INNATE (NATURAL) None ADAPTIVE (ACQUIRED) Present

immunology

Study of the immune system; the reaction of tissues of the immune system of the body to antigenic stimulation.

Nursing Assessment of a Patient With HIV Infection

Subjective Data Important Health Information Past health history: Route of infection, risk factors, history of hepatitis or other sexually transmitted infections, frequent viral infections, parasitic infections, tuberculosis, alcohol and drug use, foreign travel Medications: Use of immunosuppressive drugs Functional Health Patterns Health perception-health management: Chronic fatigue, malaise, weakness Nutritional metabolic: Unexplained weight loss; low-grade fevers, night sweats; anorexia, nausea, vomiting; oral lesions, bleeding, ulcerations; abdominal cramping; lesions of lips, mouth, tongue, throat; sensitivity to acidic, salty, or spicy foods; problems with teeth or bleeding gums; difficulty swallowing; skin rashes or color changes, lesions (painful or nonpainful), blisters; nonhealing wounds, pruritus Elimination: Persistent diarrhea, constipation, painful urination Activity-exercise: Muscle weakness, difficulty with ambulation; cough, shortness of breath Cognitive-perceptual: Headaches, stiff neck, chest pain, rectal pain, retrosternal pain; blurred vision, photophobia, loss of vision, diplopia; confusion, forgetfulness, attention deficit, changes in mental status, memory loss; hearing impairment, personality changes, paresthesias; hypersensitivity in feet Sexuality-reproductive: Lesions on genitalia (internal or external), pruritus, or burning in vagina or on penis; painful sexual intercourse; changes in menstruation; vaginal or penile discharge Objective Data General: Vital signs, weight, general status, diaphoresis Eyes: Exudate, retinal lesions or hemorrhage, papilledema, pupillary response, extraocular muscle movements Oral: A variety of mouth lesions, including blisters (herpes simplex virus type-1 lesions), white-gray patches (Candida organisms), painless white lesions on lateral aspects of tongue (oral hairy leukoplakia), discolorations (Kaposi's sarcoma), gingivitis, tooth decay or loosening Neck: Enlarged lymph nodes, nuchal rigidity, enlarged thyroid Throat: Redness or white patchy lesions Integumentary: Skin integrity and skin turgor; general appearance; lesions, eruptions, discolorations; enlarged lymph nodes, bruises, cyanosis, dryness, delayed wound healing, alopecia Respiratory: Crackles or rhonchi, dyspnea, cough (productive or nonproductive, color and amount of sputum), wheezing, tachypnea, intercostal retractions, use of accessory muscles Lymphatic: Generalized lymphadenopathy Abdominal: Tenderness, masses, enlarged liver or spleen, hyperactive bowel sounds Genitourinary-rectal: Lesions or discharge, abdominal pain indicative of pelvic inflammatory disease, difficult or painful urination Neuromuscular: Aphasia, ataxia, lack of coordination, sensory loss, tremors, slurred speech, memory loss, apathy, agitation, social withdrawal or isolation, pain, inappropriate behavior, changes in level of consciousness, depression, seizures, paralysis, coma HIV, Human immunodeficiency virus.

Dysregulation and dysfunction of which cells contribute to the immune dysfunction in HIV disease? (Select all that apply.)

T-helper cell CD4+ lymphocytes

innate immunity

The body's first line of defense; provides physical and chemical barriers to invading pathogens and protects the body against the external environment.

seroconversion

The development of detectable levels of antibodies; a change in serologic tests (e.g., ELISA and Western blot) from negative to positive as antibodies develop in reaction to an infection.

Review of Mechanisms of the Immune Response

The first time an antigen enters the body, the antigen is processed by macrophages and presented to lymphocytes. Responses of B cells to the antigen in humoral immunity require interaction with T-helper cells, which assist B cells to respond to the antigen by proliferating, synthesizing, and secreting the appropriate antibody. Antigens then are neutralized by antibodies or can form immune complexes or be phagocytosed by macrophages or neutrophils.

What factor increases the probability of vertical transmission of an HIV infection?

The mother is in the initial stage of the HIV infection.

seroprevalence

The overall frequency of findings of a disorder in a population as determined by blood testing.

Therapeutic Management

Therapeutic management of HIV-infected patients focuses on monitoring HIV disease progression and immune function, preventing the development 4803 of opportunistic diseases, initiating and monitoring ART, detecting and treating opportunistic diseases, managing symptoms, and preventing complications of treatment.

how many stages of HIV

There are three stages of HIV

Home test kits for HIV

These kits are available and the individual needs to follow the specific instructions.

Stage 1 HIV

This occurs within 2 to 4 weeks of transmission of the infection and manifests as if the patient had influenza. During this stage, patients are very infectious with elevated numbers of the virus in their blood and some bodily fluids.

Transplant Rejection

Transfer of healthy tissue or organs from a donor to a recipient has been done for many years. The immune process that protects the body from foreign protein is the same process at work in tissue transplant rejection.

vertical transmission

Transmission of HIV from a mother to a fetus; can occur during pregnancy, during delivery, or through postpartum breastfeeding.

Medical Management for Allergies

Treatment of hypersensitivity disorders includes the following: • Symptom management with medications • Environmental control • Immunotherapy The most effective treatment is environmental control,

Types of Latex Allergies

Two types of latex allergies that can occur are type IV allergic contact dermatitis and type I allergic reactions.

Latex allergy- Type IV contact dermatitis

Type IV contact dermatitis is caused by the chemicals used in the manufacturing of latex gloves. It is a delayed reaction that occurs within 6 to 48 hours. Typically the person first has dryness, pruritus, fissuring, and cracking of the skin, followed by erythema, edema, and crusting at 24 to 48 hours. Chronic exposure can lead to thickening and hardening of the skin, scaling, and hyperpigmentation. The dermatitis may extend beyond the area of physical contact with the allergen.

Oral testing for HIV

a. Uses a device that is placed 2633 against the gum and cheek for 2 minutes b. Fluid (not saliva) is drawn into an absorbable pad, which, in an HIV-positive individual, contains antibodies. c. The pad is placed in a solution and a specified observable change is noted if the test result is positive. d. If the result is positive, a blood test is needed to confirm the results.

Transfusion Reactions

are hypersensitivity disorders, best illustrated by reactions that occur with mismatched blood. They are labeled mild, moderate, and severe. The most severe reactions occur within the first 15 minutes, moderate reactions occur 4752 within 30 to 90 minutes, and mild reactions may be delayed to late in the transfusion or hours to several days after transfusion

Which immune disorder results from a failure to tolerate "self"? 1. immunodeficiency disorders. 2. hypersensitivity disorders. 3. desensitization disorders. 4. autoimmune disorders

autoimmune disorders

Attacks on beneficial foreign tissue

include organ transplant rejection and transfusion reactions.

Stage 2 HIV

known as clinical latency. The patient is often asymptomatic during this stage with low levels of the virus present in their blood.

Tissue matching

leads to a better chance of success.

Delayed Hypersensitivity

occurring 24 to 72 hours after exposure, are mediated by T cells accompanied by release of lymphokines. Delayed reaction contact dermatitis, such as after contact with poison ivy, is one example. Another example is tissue transplant rejection.

physical examination for Allergy

should include a thorough assessment of the skin, the middle ear, the conjunctiva, the naso-oropharynx, and the lungs.

Graft rejection

slowed through the use of chemical agents that interfere with the immune response. These chemical agents include corticosteroids, cyclosporine (Neoral, Sandimmune), and azathioprine (Imuran). Administration of these chemical agents is referred to as immunosuppressive therapy—the administration of agents that significantly interfere with the immune system's ability to respond to antigenic stimulation.

The patient asks the nurse, "How does HIV cause AIDS?" What is the nurse's best response? 1. "HIV attacks the immune system, a system that protects the body from foreign invaders, and thus makes it unable to protect the body from organisms that cause diseases." 2. "HIV breaks down the circulatory system, which makes the body unable to assimilate oxygen and nutrients." 3. "HIV attacks the respiratory system, which makes the lungs more susceptible to organisms causing pneumonia." 4. "HIV attacks the digestive system, which causes a decrease in the absorption of essential nutrients and results in weight loss and fatigue."

1. "HIV attacks the immune system, a system that protects the body from foreign invaders, and thus makes it unable to protect the body from organisms that cause diseases."

Saquinavir is prescribed for the client who is diagnosed with human immunodeficiency virus (HIV) seropositive. The nurse should reinforce medication instructions about which health care measure to the client? 1. Avoid sun exposure. 2. Eat low-calorie foods. 3. Eat foods that are low in fat. 4. Take the medication on an empty stomach.

1. Avoid sun exposure.

The client who is diagnosed with human immunodeficiency virus (HIV) seropositive has been taking stavudine. The nurse should monitor which parameter closely while the client is taking this medication? 1. Gait 2. Appetite 3. Level of consciousness 4. Hemoglobin and hematocrit blood levels

1. Gait

What is responsible for causing the majority of HIV disease and AIDS? 2. Human immunodeficiency virus type 2 3. African immunodeficiency virus type 1 4. Pneumocystis jiroveci deficiency virus

1. Human immunodeficiency virus type 1

The nurse is assisting with planning the care of a client with a diagnosis of immunodeficiency. The nurse should incorporate which intervention as a priority in the plan of care? 1. Protecting the client from infection 2. Providing emotional support to decrease fear 3. Encouraging discussion about lifestyle changes 4. Identifying factors that decreased the immune function

1. Protecting the client from infection

The client diagnosed with acquired immunodeficiency syndrome (AIDS) is taking nevirapine. The nurse should monitor for which side/adverse effects of the medication? Select all that apply. 1. Rash 2. Hepatotoxicity 3. Hyperglycemia 4. Peripheral neuropathy 5. Reduced bone mineral density

1. Rash 2. Hepatotoxicity

Anti-dsDNA antibody test

1. The anti-dsDNA (double-stranded DNA) antibody test is a blood test done specifically to identify or differentiate DNA antibodies found in SLE. 2. The test supports a diagnosis, monitors disease activity and response to therapy, and establishes a prognosis for SLE.

immune system has three main functions

1. To protect the body's internal environment against invading microorganisms by destroying foreign antigens and pathogens, thus preventing the development of infections

The health care provider asks the nurse to talk with a patient about how HIV is transmitted. Which routes of transmission are most important for the nurse to discuss with the patient? 1. Receiving blood, donating blood 2. Food, water, air 3. Sexual intercourse, sharing needles, mother-to-child transmission 4. Dirty toilets, swimming pools, mosquitoes

3. Sexual intercourse, sharing needles, mother-to-child transmission

What is the purpose of performing a viral load study once every 3 to 4 months in an HIV-positive person? 1. To determine the CD4 + lymphocyte count 2. To determine the progression of the disease 3. To determine the effectiveness of the medication regimen 4. To determine the results of the Western blot test

3. To determine the effectiveness of the medication regimen

immune system has three main functions

3. To serve as a surveillance network for recognizing and guarding against the development and growth of abnormal cells. Abnormal cells (mutations) are being formed constantly in the body but are recognized as abnormal cells and are destroyed

Which is another term for desensitization? 1. autoimmune disorders. 2. adaptive immunity. 3. immunotherapy. 4. immunodeficiency disease.

3. immunotherapy.

The client with diagnosed acquired immunodeficiency syndrome (AIDS) and Pneumocystis jiroveci infection has been receiving pentamidine. The client develops a temperature of 101° F (38.3° C). The nurse continues to monitor the client, knowing that this sign would most likely indicate which condition? 1. The dose of the medication is too low. 2. The client is experiencing toxic effects of the medication. 3. The client has developed inadequacy of thermoregulation. 4. A result of another infection caused by the leukopenic effects of the medication

4. A result of another infection caused by the leukopenic effects of the medication

The nurse gave an intramuscular penicillin injection to a patient. What assessment finding would be most indicative of a systemic anaphylactic response? 1. Increased blood pressure 2. Bradycardia 3. Urticaria 4. Wheezing

4. Wheezing

Medications for Hypersensitivity Reactions-diphenhydramine (Benadryl)

ACTION-Antihistamine SIDE EFFECTS- Drowsiness, confusion, nasal stuffiness, dry mouth, photosensitivity, urine retention. NURSING IMPLICATIONS- Use cautiously with central nervous system depressants, including alcohol; tell patient to avoid driving or hazardous activity because of drowsiness.

Medications for Hypersensitivity Reactions-dexamethasone

ACTION-Corticosteroid SIDE EFFECTS-Anxiety, nausea, acne, easy bruising, increased appetite. NURSING IMPLICATIONS- Do not use for extended period; use cautiously with patients with diabetes or peptic ulcers

acquired immunodeficiency syndrome AIDS

Acquired condition that impairs the body's ability to fight infection; the end stage of the continuum of HIV infection, in which the infected person has a CD4 + [lymphocyte] count of ≤200 cells/mm3

cellular immunity

Acquired immunity characterized by the dominant role of small T lymphocytes. Also called cell-mediated immunity.

hypersensitivity

An abnormal condition characterized by an excessive reaction to a particular stimulus.

human immunodeficiency virus HIV

An obligate virus; a retrovirus that causes AIDS.

More than one Autoimmune diseases.

Autoimmune diseases tend to cluster, so a given person may have more than one autoimmune disease (e.g., rheumatoid arthritis and Addison's disease). The same or related autoimmune diseases may be found in other members of the family. This observation has led to the concept of genetic predisposition to autoimmune disease.

Human immunodeficiency virus (HIV) testing

CD4 + T-cell count CD4-to-CD8 ratio Viral culture Viral load testing The p24 antigen assay Oral testing for HIV Home test kits for HIV

Innate (Natural) and Adaptive (Acquired) Immunity

CHARACTERISTICS INNATE "NATURAL" Physical barriers Physical defense: Skin and mucous membranes Mucous membranes line body cavities such as the mouth and stomach. These cavities secrete chemicals (saliva and hydrochloric acid) that destroy bacteria. Cilia, tears, and flora of the intestine and vagina also provide natural protection ADAPTIVE "ACQUIRED" None

Immunosuppressants

Calcineurin Inhibitors Cyclosporine Tacrolimus Cytotoxic Medications 2676 Azathioprine Cyclophosphamide Methotrexate Mycophenolate mofetil Mycophenolic acid Antibodies Basiliximab Lymphocyte immune globulin, antithymocyte globulin Muromonab-CD3 Rh0 (D) immune globulin Other Sirolimus Everolimus Glucocorticoids

Pneumocystis jiroveci

Cases were bein identified as 1979, physicians in New York and California (formerly Pneumocystis carinii) pneumonia, an unusual pulmonary disease caused by a fungus that infects primarily people who have suppressed immune systems. The fungus is found commonly in the environment but does not normally affect people who are healthy.

Innate (Natural) and Adaptive (Acquired) Immunity

Cells CHARACTERISTICS INNATE (NATURAL) Phagocytes, natural killer (NK) cells ADAPTIVE (ACQUIRED) T cells, B cells

Types of White Blood Cells and Their Involvement in HIV Disease- T helper cells (CD4+ or T4 cells)

Contain CD4 + receptors Considered the "conductor" of the immune system because of their secretion of cytokines, which control most aspects of the immune response ROLE IN HIV infections. Infection with HIV can impair T helper cell function without killing the cell.

Common Allergens Causing Anaphylaxis- FOODS

Cow's milk Peanuts Brazil nuts Cashew nuts Shellfish Egg albumin Strawberries Chocolate Soy

opportunistic

Disease characteristic caused by a normally nonpathogenic organism in a host whose resistance has been decreased by a disorder such as AIDS.

autoimmune responses

Environmental factors, including smoking, viral infections, and low vitamin D levels, have been found to trigger autoimmune diseases in susceptible people . In addition, high salt intake may be associated with the autoimmune response . Many illnesses are now believed to be in this classification. Included are pernicious anemia, Guillain-Barré syndrome, scleroderma, Sjögren's syndrome, rheumatic fever, rheumatoid arthritis, ulcerative colitis, male infertility, myasthenia gravis, multiple sclerosis, Addison's disease, autoimmune hemolytic anemia, immune thrombocytopenic purpura, type 1 diabetes mellitus, glomerulonephritis, and systemic lupus erythematosus.

Autoimmune disorders fall into which category of "inappropriate responses of the immune system?"

Failure to recognize the body as self

Latex Allergy

Description 1. Latex allergy is a hypersensitivity to latex. 2. The source of the allergic reaction is thought to be the proteins in the natural rubber latex or the various chemicals used in the manufacturing process of latex gloves. 3. Symptoms of the allergy can range from mild contact dermatitis to moderately severe symptoms of rhinitis, conjunctivitis, urticaria, and bronchospasm to severe lifethreatening anaphylaxis. B. Common routes of exposure (Ace bandages (brown) Adhesive or elastic bandages Ambu bag Balloons Blood pressure cuff (tubing and bladder) Catheter leg bag straps Catheters Condoms Diaphragms Elastic pressure stockings Electrocardiogram pads Feminine hygiene pads Gloves Intravenous catheters, tubing, and rubber injection ports Nasogastric tubes Pads for crutches Prepackaged enema kits Rubber stoppers on medication vials Stethoscopes Syringe ) 1. Cutaneous: Natural latex gloves and latex balloons 2. Percutaneous and parenteral: Intravenous lines and catheters; hemodialysis equipment 3. Mucosal: Use of latex condoms, catheters, airways, and nipples 4. Aerosol: Aerosolization of powder from latex gloves can occur when gloves are dispensed from the box or when gloves are removed from the hands. C. At-risk individuals 1. Health care workers 2. Individuals who work in the rubber industry 3. Individuals having multiple surgeries 4. Individuals with spina bifida 5. Individuals who wear gloves frequently such as food handlers, hairdressers, and auto mechanics 6. Individuals allergic to kiwis, bananas, pineapples, tropical fruits, grapes, avocados, potatoes, hazelnuts, and water chestnuts D. Data collection 1. Anaphylaxis or type I hypersensitivity is a response to natural rubber latex (Figure 59-2; also see Figure 59-1). 2. A delayed type IV hypersensitivity can occur within 6 to 48 hours. Symptoms of contact dermatitis include pruritus, edema, erythema, vesicles, papules, and crusting and thickening of the skin. E. Interventions (Box 59-2)

immune system failure

Failures may be the result of genetic factors, developmental defects, infection, malignancy, injury, drugs, or altered metabolic states.

cocktails

Founded by scientist to be be the most effective medication regime. (generally three or more compounds given together).

Which statement most accurately describes transmission of HIV?

HIV is transmitted from human to human via infected blood, semen, rectal secretions, cervicovaginal secretions, and breast milk.

Review of Mechanisms of the Immune Response

Immunodeficiency is an abnormal condition of the immune system in which cellular or humoral immunity is inadequate and resistance to infection is decreased. The immunodeficiency diseases sometimes are classified as B cell (antibody) deficiencies, T cell (cellular) deficiencies, and combined T and B cell deficiencies.

The nurse is caring for a patient who has had an organ transplant. The patient inquires about rejection and medications used to prevent it. On what knowledge will the nurse's response be based? (Select all that apply.)

Immunosuppressive therapy is helpful in slowing the process of graft rejection. Antigenic determinants on the cells lead to graft rejection via the immune process Infection is a threat to the patient receiving immunosuppressive therapy.

What is the technique of assisting the body to develop immunity by way of injecting a diluted antigen, in a series of injections of increasing strength, over a 1- to 3-year period?

Immunotherapy

Recognize self from nonself

Normally the body recognizes its own cells and does not produce antibodies to fight those cells; therefore an immune response generally is triggered only in response to agents that the body identifies as foreign. Autoimmune disorders disrupt the ability to differentiate self from nonself, and the immune system attacks the body's own cells as if they were foreign antigens.

Medications for Human Immunodeficieny Virus (HIV) and Acquired Immunodeficiency Syndrome (AIDS)

Nucleoside-Nucleotide Reverse Transcriptase Inhibitors (NRTIs) Abacavir Abacavir/lamivudine Didanosine Emtricitabine Emtricitabine/tenofovir Emtricitabine/tenofovir/efavirenz Lamivudine Lamivudine/zidovudine Lamivudine/zidovudine/abacavir Stavudine Tenofovir Zidovudine Nonnucleoside Reverse Transcriptase Inhibitors (NNRTIs) Delavirdine Efavirenz Etravirine Nevirapine Protease Inhibitors (PIs) Atazanavir Danunavir Fosamprenavir Indinavir Lopinavir/ritonavir Nelfinavir Ritonavir Saquinavir Tipranavir Integrase Inhibitor Raltegravir Dolutegravir Elvitegravir Fusion Inhibitor Enfuvirtide 2675 Chemokine Receptor 5 (CCR5) Antagonist Maraviroc Antiinflammatory Medication Sulfasalazine Antiinfective Medications Atovaquone Metronidazole Pentamidine isethionate Sulfamethoxazole/trimethoprim Antifungal Medications Amphotericin B Fluconazole Ketoconazole Itraconazole Voriconazole Antiviral Medications Acyclovir Foscarnet Ganciclovir Valacyclovir

B cells

Originate and processed in the red marrow. produce antibodies to destroy specific foreign antigens (humoral immunity). make up approximately 20% to 30% of the lymphocyte population. trigger the production of antibodies and proliferate (increase in number) in response to a particular antigen (a substance recognized by the body as foreign and that can trigger an immune response). migrate to the peripheral circulation and tissues and eventually are filtered from the lymph and stored in the lymphoid tissue of the body. The initial formation of B cells does not require antigen stimulation or any other environmental stimulus. However, B cell proliferation depends on antigen stimulation. B cells are responsible for humoral immunity. B cells produce antibodies and protect against bacteria, viruses, and soluble antigens.

What cells are responsible both for cell-mediated (cellular) immunity and protection of the body against viruses?

T cells

Palliative Care

The World Health Organization (WHO) defines palliative care as helping patients and families deal with possibly fatal illnesses and quality-of-life issues. The focus of palliative care is on preventing and relieving suffering by quickly identifying and treating p

immunocompetence

The ability of an immune system to mobilize and deploy its antibodies and other responses to stimulation by an antigen.

Which statement is most accurate regarding diagnostic tests for HIV and AIDS?

The ability to detect HIV viral load measurements in plasma is a significant advancement in the monitoring of HIV disease.

Hypersensitivity Disorders (continue)

The host becomes sensitive after the first exposure and on subsequent exposure exhibits a hypersensitivity reaction. Chronic exposure leads to a chronic allergy response, which ranges from mild to incapacitating signs and symptoms. Hypersensitivity disorders are believed to be caused by a genetic defect that allows increased production of immunoglobulin E (IgE) with release of histamine and other mediators from mast cells and basophils

Respond to nonself invaders

The immune system responds by producing antibodies that target specific antigens for destruction. New antibodies are produced in response to new antigens. Deficits in the ability to respond may be the result of immunodeficiency disorders.

Immunodeficiency disorders involve

an impairment of one or more of the following immune mechanisms: • Phagocytosis • Humoral response • Cell-mediated response • Complement response • Combined humoral and cell-mediated deficiency

best method for preventing transfusion reaction

autologous transfusion i.e., use of one's own blood for replacement therapy. The blood can be frozen and stored for as long as 3 years. Usually the blood is stored without being frozen and is given to the person within a few weeks of donation.

Allergies

caused by hyperactive responses against environmental antigens. This would include medication allergies as well as allergies to allergens such as dust, mold, and pet dander.

Immunoglobulin levels

decrease with age and therefore lead to a suppressed humoral immune response in older adults.

Mild transfusion reaction signs and symptoms

include dermatitis, diarrhea, fever, chills, urticaria, cough, and orthopnea. Treatment includes (1) stopping the transfusion; (2) notifying the health care provider; and (3) administering saline, steroids, and diuretics as ordered. The health care provider may give instructions to restart the transfusion at a slower rate and to continue to monitor the patient.

Other examples of zoonotic transmission

include severe acute respiratory distress syndrome, anthrax, rabies, hantavirus, and West Nile virus.

Autoimmune disorders

include systemic lupus erythematosus, celiac disease, thyroid disease, inflammatory bowel disease, type 1 diabetes, multiple sclerosis, myasthenia gravis, psoriasis, rheumatoid arthritis, and many other common diseases.

sub-classifications of immunity

innate [natural]- nonspecific adaptive [acquired]- specific

Thymic involution (i.e., shrinking of the thymus)

occurs with aging, along with decreased numbers of T cells. The incidence of malignancies and autoimmune diseases increases with aging and may be related to immunologic deterioration.

The p24 antigen assay

quantifies the amount of HIV viral core protein in the client's serum.

Factors Influencing Hypersensitivity

• Host response to allergen: The more sensitive the individual, the greater the allergic response. • Exposure amount: In general, the more allergen the individual is exposed to, the greater the chance of a severe reaction. • Nature of the allergen: Most allergic reactions are precipitated by complex, high-molecular-weight protein substances. • Route of allergen entry: Most allergens enter the body via gastrointestinal and respiratory routes. Exposure to venoms through bites or stings and injectable medications present a more severe threat of allergic response. • Repeated exposure: In general, the more often the individual is exposed to the allergen, the greater the response.

The Four R's of the Immune Response

• Recognize self from nonself. • Respond to nonself invaders. • Remember the invader. • Regulate its action

4 category of inappropriate immune responses

1. Allergies 2. Immunodeficiency 3. Autoimmune disorders 4. Attacks on beneficial foreign tissue

disorders categories

1. Hypersensitivity disorder involves allergic response and tissue rejection. 2. Immunodeficiency disease involves altered and failed immune response. 3. Autoimmune disease involves extensive tissue damage resulting from an immune system that seemingly reverses its function to one of self-destruction.

The patient tells the nurse he is overwhelmed. There is so much he must do to keep his new kidney functioning, and then rejection may still occur. Which patient problem statement is appropriate? 1. Impaired Coping 2. Distorted Body Image 3. Potential for Unsafe Health Behaviors 4. Impaired Self-Esteem due to Current Situation

1. Impaired Coping

The camp nurse prepares to instruct a group of children about Lyme disease. Which information should the nurse include in the instructions? 1. Lyme disease is caused by a tick carried by deer. 2. Lyme disease is caused by contamination from cat feces. 3. Lyme disease can be contagious by skin contact with an infected individual. 4. Lyme disease can be caused by the inhalation of spores from bird droppings.

1. Lyme disease is caused by a tick carried by deer.

How does vertical transmission of HIV occur? (Select all that apply.) 1. Mother to fetus during breast-feeding 2. Mother to fetus during pregnancy 3. Mother to fetus during delivery 4. Mother to fetus during the postpartum period 5. Mother to fetus during the first year of the infant's life

1. Mother to fetus during breast-feeding 2. Mother to fetus during pregnancy 3. Mother to fetus during delivery

The patient is concerned that he may be infected with HIV. Which sign or symptom is most commonly associated with an HIV infection? 1. Night sweats 2. Rash on the legs only 3. Constipation 4. Pruritus

1. Night sweats

Goals for the National HIV Strategy

1. Reduce the number of new HIV infections. 2. Increase access to care and improve health outcomes for people living with HIV. 3. Reduce HIV-related health disparities and health inequities. 4. Achieve a more coordinated national response to the HIV epidemic. The following are indicators of the progress of this strategy: • Increase the percentage of people living with HIV who know their serostatus to at least 90%. • Reduce the number of new diagnoses of HIV by at least 25%. • Reduce the percentage of young gay and bisexual men who have engaged in HIV-risk behaviors by at least 10%. • Increase the percentage of newly diagnosed persons linked to HIV medical care within 1 month of their HIV diagnosis to at least 85%. • Increase the percentage of persons with diagnosed HIV infection who are retained in HIV medical care to at least 90%. • Increase the percentage of persons with diagnosed HIV infection who are virally suppressed to at least 80%. • Reduce the percentage of persons in HIV medical care who are homeless to no more than 5%. • Reduce the death rate among persons with diagnosed HIV infection by at least 33%. • Reduce disparities in the rate of new diagnoses by at least 15% in the following groups: gay and bisexual men, young black gay and bisexual men, black females, and persons living in the southern United States. • Increase the percentage of youth and persons who inject drugs with diagnosed HIV infection who are virally suppressed to at least 80%. aFrom HIV.gov: What is the national HIV/ AIDs strategy.

Which statements describe innate, or natural, immunity? (Select all that apply.) 1. The body's first line of defense against disease, which protects locally against the external environment 2. The body's second line of defense against disease, which protects the internal environment 3. Includes stomach acid, saliva, and secretions/flora of the intestine and vagina 4. Mediated by B cells to produce antibodies in response to antigenic challenge 5. Provides an immunity that is specific 6. Responds with neutrophils, macrophages, and lymphocytes

1. The body's first line of defense against disease, which protects locally against the external environment 3. Includes stomach acid, saliva, and secretions/flora of the intestine and vagina 6. Responds with neutrophils, macrophages, and lymphocytes

Which interventions should be implemented in the care of a client at high risk for an allergic response to a latex allergy? Select all that apply. 1. Use nonlatex gloves. 2. Use medications from glass ampules. 3. Place the client in a private room only. 4. Do not puncture rubber stoppers with needles. 5. Keep a latex-safe supply cart available in the client's area. 6. Use a blood pressure cuff from an electronic device only to measure the blood pressure.

1. Use nonlatex gloves. 2. Use medications from glass ampules. 4. Do not puncture rubber stoppers with needles. 5. Keep a latex-safe supply cart available in the client's area.

HIV Life Cycle

1. binding-(attached to itself) to receptors on the surface of a CD4 cell. CCRS antagonists, Post-attachment inhibitors) 2. fusion- HIV envelope and the CD4 cell membrane fuse together, which allows HIV to enter the CD4 cell. (Fusion inhibitors) 3. reverse transcription- inside the CD4 cell, HIV releases and uses reverse transcription (an HIV enzyme) to convert its genetic material-HIV RNA-into HIV DNA. The conversion of HIV RNA to DNA allows HIV to enter the CD4 cell nucleus and combine with the cell's genetic material-cell DNA). 4. integration- (inside the CD4 cell nucleus, HIV releases integrase (an HIV enzyme). HIV uses integrase to insert (integrate) its viral DNA into the DNA of the CD4 cell. 5. replication- Once integrated into the CD4 cell DNA, HIV begins to use the machinery of the CD4 cell to make long chains of HIV proteins. The protein chain are the building block for more HIV. 6. assembly-New HIV proteins and HIV RNA move to the surface of the cell and assemble into immature (noninfectious) HIV. 7. budding- Newly formed immature (noninfectious) HIV pushes itself out of the host CD4 cell. The new HIV releases protease (an HIV enzyme). Protease acts to break up the long protein chains that form the immature virus. The smaller HIV proteins combine to form mature (infectious) HIV.

An 8-year-old boy receives a diagnosis of hemophilia. His mother is upset about his risk of acquiring HIV from blood products. What would be the nurse's best response? 1. "All blood and blood products are screened for bloodborne diseases, so it is impossible for your son to be infected." 2. "Many blood products are treated with heat or chemicals to inactivate the HIV virus." 3. "We can talk about this if your son requires transfusions or blood products." 4. "All blood donors are asked about HIV status and risk factors before giving blood."

2. "Many blood products are treated with heat or chemicals to inactivate the HIV virus."

The activation of T cells by antigens indicate which type of immunity is developing? (Select all that apply.) 1. Innate immunity 2. Adaptive immunity 3. Humoral immunity 4. Cellular immunity

2. Adaptive immunity 4. Cellular immunity

The client diagnosed with pemphigus is being seen in the clinic regularly. The nurse should plan care based on which description of this condition? 1. The presence of tiny red vesicles 2. An autoimmune disease that causes blistering in the epidermis 3. The presence of skin vesicles found along the nerve caused by a virus 4. The presence of red, raised papules and large plaques covered by silvery scales

2. An autoimmune disease that causes blistering in the epidermis

The client calls the office of the primary health care provider (PHCP) and states to the nurse that they were just stung by a bumblebee while gardening. The client is afraid of a severe reaction because their neighbor experienced such a reaction just 1 week ago. Which should be the appropriate nursing action? 1. Advise the client to soak the site in hydrogen peroxide. 2. Ask the client if they ever sustained a bee sting in the past. 3. Tell the client to call an ambulance for transport to the emergency room. 4. Tell the client not to worry about the sting unless difficulty with breathing occurs.

2. Ask the client if they ever sustained a bee sting in the past.

A patient experienced an anaphylactic reaction to an antibiotic. What are the correct nursing interventions for anaphylaxis? (Select all that apply.) 1. Assess vital signs every 4 hours. 2. Assess respiratory status frequently. 3. Maintain patent airway. 4. Administer epinephrine as ordered. 5. Monitor vital signs frequently

2. Assess respiratory status frequently. 3. Maintain patent airway. 4. Administer epinephrine as ordered. 5. Monitor vital signs frequently

The nurse is caring for a patient with a history of numerous allergies. What is the most important teaching concept? 1. Immunotherapy regimen 2. Avoidance of the allergen 3. Antihistamine administration 4. Adrenaline administration

2. Avoidance of the allergen

Humoral immunity is mediated by which type of cells? 1. T cells 2. B cells 3. Macrophages 4. Myeloblasts

2. B cells

The most common allergens that cause an anaphylactic reaction include: (Select all that apply.) 1. Leafy, green vegetables 2. Bees, wasps 3. Shellfish 4. Peanuts 5. Oranges

2. Bees, wasps 3. Shellfish 4. Peanuts

The client is diagnosed with stage I of Lyme disease. The nurse should check the client for which characteristic of this stage? 1. Arthralgia 2. Flu-like symptoms 3. Enlarged and inflamed joints 4. Signs of a neurological disorder

2. Flu-like symptoms

The client brought to the emergency department is experiencing an anaphylactic reaction from eating shellfish. The nurse should implement which immediate action? 1. Administering epinephrine 2. Maintaining a patent airway 3. Administering a corticosteroid 4. Instructing the client on the importance of obtaining a MedicAlert bracelet

2. Maintaining a patent airway

Ketoconazole is prescribed for a client with a diagnosis of candidiasis. Which interventions should the nurse include when administering this medication? Select all that apply. 1. Restrict fluid intake. 2. Monitor liver function studies. 3. Instruct the client to avoid alcohol. 4. Administer the medication with an antacid. 5. Instruct the client to avoid exposure to the sun. 6. Administer the medication on an empty stomach.

2. Monitor liver function studies. 3. Instruct the client to avoid alcohol. 5. Instruct the client to avoid exposure to the sun.

What are the most common opportunistic infection and malignant neoplasm in the patient with advanced HIV disease (AIDS)? 1. Streptococcal pneumonitis, myeloma 2. Pneumocystis jiroveci pneumonia, Kaposi's sarcoma 3. Streptococcus pneumoniae, malignant melanoma 4. Mycoplasma pneumonitis, Kaposi's sarcoma

2. Pneumocystis jiroveci pneumonia, Kaposi's sarcoma

The student nurse is giving a presentation on the transmission of HIV to his class. The student is correct by sharing which information? (Select all that apply.) 1. Health care workers are at a high risk for contracting HIV. 2. Risk for transmission of HIV is higher with anal intercourse than with other types of sexual intercourse. 3. People who use injection drugs and share needles have an elevated risk for being infected with HIV. 4. The leading mode of transmission of HIV worldwide is sexual intercourse, despite sexual preference. 5. During sexual intercourse, the partner who is being penetrated is least likely to contract HIV.

2. Risk for transmission of HIV is higher with anal intercourse than with other types of sexual intercourse. 3. People who use injection drugs and share needles have an elevated risk for being infected with HIV. 4. The leading mode of transmission of HIV worldwide is sexual intercourse, despite sexual preference.

For most people who are HIV positive, marker antibodies are usually present 10 to 12 weeks after exposure. What is the development of these antibodies called? 1. Immunocompetence 2. Seroconversion 3. Immunodeficiency 4. Viral load

2. Seroconversion

immune system has three main functions

2. To maintain homeostasis by removing damaged cells from the circulation, thereby maintaining the body's various cell types in an unchanged and uniform form

The nurse prepares to give a bath and change the bed linens for a client with cutaneous Kaposi's sarcoma lesions. The lesions are open and draining a scant amount of serous fluid. Which should the nurse incorporate in the plan during the bathing of this client? 1. Wearing gloves 2. Wearing a gown and gloves 3. Wearing a gown, gloves, and a mask 4. Wearing a gown and gloves to change the bed linens and gloves only for the bath

2. Wearing a gown and gloves

The nurse, a Cub Scout leader, is preparing a group of Cub Scouts for an overnight camping trip and instructs them about the methods to prevent Lyme disease. Which statement by one of the Cub Scouts indicates a need for further teaching? 1. "I need to bring a hat to wear during the trip." 2. "I should wear long-sleeved tops and long pants." 3. "I should not use insect repellent because it will attract the ticks." 4. "I need to wear closed shoes and socks that can be pulled up over my pants."

3. "I should not use insect repellent because it will attract the ticks."

The nurse is caring for a postrenal transplantation client with prescription for cyclosporine. If the nurse notes an increase in one of the client's vital signs and the client is complaining of a headache, which vital sign is most likely increased? 1. Pulse 2. Respirations 3. Blood pressure 4. Pulse oximetry

3. Blood pressure

A patient comes to the clinic for his weekly allergy injection. He missed his appointment the week before because of a family emergency. Which action is appropriate in administering the patient's injection? 1. Administer the usual dose of the allergen. 2. Double the dose to account for the missed injection the previous week. 3. Consult with the health care provider about decreasing the dose for this injection. 4. Reevaluate the patient's sensitivity to the allergen with a skin test.

3. Consult with the health care provider about decreasing the dose for this injection.

After a bee sting, the patient's face becomes edematous and she begins to wheeze. Based on this assessment, the nurse would be prepared to administer which medication? 1. Aminophylline 2. Diphenhydramine 3. Epinephrine 4. Diazepam

3. Epinephrine

Amikacin is prescribed for a client with a diagnosed bacterial infection. The nurse instructs the client to contact the primary health care provider (PHCP) immediately if which occurs? 1. Nausea 2. Lethargy 3. Hearing loss 4. Muscle aches

3. Hearing loss

What diet should the patient with HIV follow? 1. High calorie, high fiber, low protein 2. Low calorie, low fiber, high protein 3. High calorie, high protein, low residue 4. Low calorie, high fiber, high protein

3. High calorie, high protein, low residue

A patient is undergoing plasmapheresis for treatment of systemic lupus erythematosus. The nurse correctly recognizes which statement about the procedure? (Select all that apply.) 1. Plasmapheresis will be used to remove the T lymphocytes from the blood that are responsble for producing antinuclear antibodies. 2. Plasmapharesis will remove normal particles in her blood that are being damaged by autoantibodies. 3. Plasmapharesis will replace the plasma that contains antinuclear antibodies with a substitute fluid. 4. Plasma removed during the procedure will be replaced by normal saline. 5. A side effect of the procedure is hypertension.

3. Plasmapharesis will replace the plasma that contains antinuclear antibodies with a substitute fluid. 4. Plasma removed during the procedure will be replaced by normal saline.

The client with acquired immunodeficiency syndrome (AIDS) is diagnosed with cutaneous Kaposi's sarcoma. Based on this diagnosis, the nurse should determine that this has been confirmed by which finding? 1. Swelling in the genital area 2. Swelling in the lower extremities 3. Punch biopsy of the cutaneous lesions 4. Appearance of reddish-blue lesions on the skin

3. Punch biopsy of the cutaneous lesions

The nurse is reviewing the results of serum laboratory studies drawn on a client diagnosed with acquired immunodeficiency syndrome (AIDS) who is receiving didanosine. The nurse determines that the client may have the medication discontinued by the primary health care provider (PHCP) if which significantly elevated result is noted? 1. Serum protein 2. Blood glucose 3. Serum amylase 4. Serum creatinine

3. Serum amylase

The nurse is assisting with the administration of immunizations at a health care clinic. The nurse should understand that immunization provides which protection? 1. Protection from all diseases 2. Innate immunity from disease 3. Natural immunity from disease 4. Acquired immunity from disease

4. Acquired immunity from disease

Which individual is least at risk for the development of Kaposi's sarcoma? 1. A kidney transplant client 2. A male with a history of same-sex partners 3. A client receiving antineoplastic medications 4. An individual working in an environment where exposure to asbestos exists

4. An individual working in an environment where exposure to asbestos exists

A patient receives a diagnosis of HIV disease. She visits the physician today for her prescriptions. What medication does the nurse expect the physician to order for the patient? 1. Tenofovir and emtricitabine (Truvada) 2. Efavirenz (Sustiva) 3. Lopinavir and ritonavir (Kaletra) 4. Combination of these drugs

4. Combination of these drugs

The client diagnosed with acquired immunodeficiency syndrome (AIDS) has begun therapy with zidovudine. The nurse should monitor which laboratory result during treatment with this medication? 1. Blood culture 2. Blood glucose level 3. Blood urea nitrogen 4. Complete blood count

4. Complete blood count

The nurse is assessing a patient who has requested HIV testing. What would be considered to be the most risky behavior? 1. Dry kissing a casual date 2. Sharing a soda with an infected person 3. Swimming with an infected person 4. Having more than three sex partners in a year

4. Having more than three sex partners in a year

The client arrives at the health care clinic and states to the nurse that they were just bitten by a tick and would like to be tested for Lyme disease. The client tells the nurse that they removed the tick and flushed it down the toilet. Which nursing action is appropriate? 1. Refer the client for a blood test immediately. 2. Inform the client that there is not a test available for Lyme disease. 3. Tell the client that testing is not necessary unless arthralgia develops. 4. Instruct the client to return in 4 to 6 weeks to be tested, because testing before this time is not reliable.

4. Instruct the client to return in 4 to 6 weeks to be tested, because testing before this time is not reliable.

The nurse is assigned to care for a client diagnosed with systemic lupus erythematosus (SLE). The nurse should plan care considering which factor regarding this diagnosis? 1. A local rash occurs as a result of allergy. 2. It is a disease caused by overexposure to sunlight. 3. A continuous release of histamine in the body causes the disease. 4. It is an inflammatory disease of collagen contained in connective tissue.

4. It is an inflammatory disease of collagen contained in connective tissue.

The client is suspected of having systemic lupus erythematous (SLE). The nurse monitors the client, knowing that which is one of the initial characteristic signs of SLE? 1. Weight gain 2. Subnormal temperature 3. Elevated red blood cell count 4. Rash on the face across the nose and on the cheeks

4. Rash on the face across the nose and on the cheeks

The nurse is assigned to care for the client diagnosed with cytomegalovirus retinitis and acquired immunodeficiency syndrome (AIDS) who is receiving foscarnet. The nurse should monitor the results of which laboratory study while the client is taking this medication? 1. CD4 + cell count 2. Lymphocyte count 3. Serum albumin level 4. Serum creatinine level

4. Serum creatinine level

What is the expanded definition of AIDS? 1. The patient has HIV infection. 2. The patient has a dysfunction of the immune system and is HIV positive. 3. The patient is HIV positive and has an opportunistic disease. 4. The patient is HIV positive with a CD4 + lymphocyte count lower than 200/mm3 or has an AIDS-related opportunistic infection.

4. The patient is HIV positive with a CD4 + lymphocyte count lower than 200/mm3 or has an AIDS-related opportunistic infection.

A 38-year-old patient is receiving 2 units of packed red blood cells at 125 mL/h. Fifteen minutes after the start of the blood transfusion, the nurse notes the following vital signs: pulse 110 beats/minute, respirations 28 breaths/minute, blood pressure 98/58 mm Hg, and temperature 101°F. The patient is shivering. What is the nurse's initial action? 1. Slow the infusion rate. 2. Stop the infusion. 3. Administer aspirin as ordered for elevated temperature. 4. Report the findings to the nurse manager.

4. Wheezing

According to the CDC what percentage of men who have sex with other men (MSM) accounted for the total number of HIV infections in males 13 years old and older?

78%

immune system

A system (including the thymus and bone marrow and lymphoid tissues) that protects the body from foreign substances and pathogenic organisms by producing the immune response

Hypersensitivity and Allergy

A. Description 1. An allergy is an abnormal, individual response to certain substances that normally do not trigger such an exaggerated reaction. 2. With some types of allergies, a reaction occurs on a second and subsequent contact with the allergen. 3. Skin testing may be done to determine the allergen. B. Data collection 1. History of exposure to allergen(s) 2. Itching, tearing, and burning of the eyes and skin 3. Rash(es) 4. Nose twitching and nasal stuffiness C. Interventions 1. Identification of the specific allergen 2. Management of the symptoms with antihistamines, antiinflammatory agents, or corticosteroids 3. Ointments, creams, wet compresses, and soothing baths for local reactions 4. Desensitization programs may be recommended.

Anaphylaxis

A. Description 1. Anaphylaxis is a serious and immediate hypersensitivity reaction with the release of histamine from the damaged cells. 2. Anaphylaxis can be systemic or cutaneous (localized). B. Data collection C. Interventions Priority Nursing Actions Anaphylaxis Reaction 1. Quickly check respiratory status and maintain a patent airway. 2. Call the registered nurse (RN), who will contact the primary health care provider (PHCP) or rapid response team. 3. Administer oxygen. 4. Assist the RN with preparing to start an intravenous (IV) line and infuse normal saline. 5. Prepare the administration of diphenhydramine and epinephrine. 6. Document the event, actions taken, and the client's response. If the client experiences an anaphylactic reaction, the immediate action would be to check the respiratory status quickly and maintain a patent airway. The nurse stays with the client and calls out for the RN, who will contact the PHCP and/or rapid response team. In the meantime, the nurse stays with the client and monitors the client's vital signs and for signs of shock. An IV device is inserted if one is not already in place, and normal saline is infused. The nurse then prepares with the assistance of the administration of diphenhydramine and epinephrine and other medications as prescribed. The head of the bed is elevated if the client's blood pressure is normal. The client's feet and legs may be raised if the blood pressure is low. The nurse documents the event, the actions taken, and the client's response.

Medications for Hypersensitivity Reactions-flunisolide

ACTION- Corticosteroid (inhaled. SIDE EFFECTS- Headache, transient nasal burning, epistaxis, nausea, vomiting. NURSING IMPLICATIONS- Not effective for acute episodes; use regularly; teach care and cleaning of inhaler; if symptoms do not improve in 3 wk, consult health care provider.

Antibiotics and their Adverse Effects

Aminoglycosides -Ototoxicity Confusion, disorientation Renal toxicity Gastrointestinal irritation Palpitations, blood pressure changes Hypersensitivity reactions Cephalosporins- Gastrointestinal disturbances Pseudomembranous colitis Headache, dizziness, lethargy, paresthesia Nephrotoxicity Superinfections Fluoroquinolones- Headache, dizziness, insomnia, depression Gastrointestinal effects Bone marrow depression Fever, rash, photosensitivity Macrolides- Gastrointestinal effects Pseudomembranous colitis Confusion Superinfections Hypersensitivity reactions Lincosamides- Gastrointestinal effects Pseudomembranous colitis Bone marrow depression Monobactams- Gastrointestinal effects Hepatotoxicity Allergic reactions Penicillins and penicillinase-resistant penicillins- Gastrointestinal effects, including sore mouth and furry tongue Superinfections Hypersensitivity reactions, including anaphylaxis Sulfonamides- Gastrointestinal effects Hepatotoxicity Nephrotoxicity Bone marrow depression Dermatological effects, including hypersensitivity and photosensitivity Headache, dizziness, vertigo, ataxia, depression, seizures Tetracyclines- Gastrointestinal effects Hepatotoxicity Teeth (staining) and bone damage Superinfections Dermatological reactions, including rash and photosensitivity Hypersensitivity reactions Antimycobacterials, leprostatics- Gastrointestinal effects Neuritis, dizziness, headache, malaise, drowsiness, hallucinations Antifungals- Gastrointestinal effects Headache, rash, anemia, hepatotoxicity, hearing loss, peripheral neuritis

Antibiotics

Aminoglycosides Amikacin Gentamicin Neomycin Streptomycin Tobramycin Antimycobacterials Antituberculosis agents (see Chapter 48) Leprostatics: Clofazimine, Thalidomide Antifungal Medications 2677 Amphotericin B Fluconazole Ketoconazole Itraconazole Voriconazole Antiviral Medications Acyclovir Foscarnet Ganciclovir Valacyclovir Cephalosporins Cefaclor Cefadroxil Cefazolin Cefdinir Cefditoren Cefepime Cefotaxime Cefotetan Cefoxitin Cefpodoxime Cefprozil Ceftazidime Ceftibuten Ceftriaxone Cefuroxime Cephalexin Fluoroquinolones Ciprofloxacin Gemifloxacin Levofloxacin Moxifloxacin Norfloxacin Ofloxacin Lincosamides Clindamycin 2678 Lincomycin Macrolides Azithromycin Clarithromycin Erythromycin Monobactam Aztreonam Penicillins Amoxicillin Ampicillin Penicillin G Penicillin V Piperacillin Penicillinase-Resistant Penicillins Dicloxacillin Nafcillin Oxacillin Sulfonamides Sulfadiazine Sulfamethoxazole Sulfasalizine Sulfisoxazole Trimethoprim/sulfamethoxazole Tetracyclines Demeclocycline Doxycycline Minocycline Tetracycline

immunodeficiency

An abnormal condition of the immune system in which cellular or humoral immunity is inadequate and resistance to infection is decreased.

Pneumocystis jiroveci pneumonia PCP

An unusual pulmonary disease caused by a parasite that is primarily associated with people who have suppressed immune systems, especially in people with AIDS.

enzyme-linked immunosorbent assay ELISA

Antibody test that uses a rapid enzyme immunochemical assay method to detect HIV antibodies.

immunogen

Any agent or substance capable of provoking an immune response or producing immunity.

Antiretroviral Therapy

Combination antiretroviral therapy (ART) is an important component in the management of HIV infection. In 1987 zidovudine was the only medication available to treat patients with HIV disease; since then, the FDA has approved more than 25 antiretroviral medications. Six different classes 4805 of ART are used to prevent the viral replication process: integrase strand transfer inhibitors, fusion inhibitors, CCR5 antagonists (also referred to as entry inhibitors), nucleoside reverse transcriptase inhibitors (NRTIs), nonnucleoside reverse transcriptase inhibitors (NNRTIs), and protease inhibitors (PI). Each class of medication interrupts HIV at different stages of the infectious process. Some ART medications are available as a combination of medications so that several medications are combined into one pill that is taken each day, which allows for better patient compliance (HIV.gov, 2017a). In ART, HIV is treated with three or more antiretroviral medications,

Types of White Blood Cells and Their Involvement in HIV Disease- Cytotoxic T cells or cytotoxic T lymphocytes (CTLs; CD8+ cells)

Contain CD8 + receptors and produce cytokines in a more limited manner than do CD4 + cells Combat viral and bacterial infections and are involved in direct killing of target cells by binding to them and releasing a substance that can perforate the cell membrane. ROLE IN HIV Numbers of these cells increase in HIV infection. This increase represents the cellular response to infection. The strength of this initial cellular response has been shown to predict progression to AIDS (i.e., better cell response implies slower disease progression). Cytotoxic T cells kill T helper cells infected with HIV.

Centers for Disease Control and Prevention (CDC)

Federal agency that provides facilities and services for investigation, identification, prevention, and control of disease; headquartered in Atlanta, Georgia.

obligate virus

HIV is an obligate virus, which means it must have a host organism to survive. The virus cannot live long outside the human body. HIV transmission depends on the presence of the virus, the infectiousness of the virus, the susceptibility of the uninfected recipient, and any conditions that may increase the recipient's risk for infection. HIV is transmitted from human to human through infected blood, semen, rectal secretions, cervicovaginal secretions, and breast milk

CLASSIFICATION OF HIV

HIV is classified as a "slow" retrovirus or a lentivirus. After infection with these types of viruses, a long time passes before specific signs and symptoms appear. HIV requires cells for replication. The virus takes over the host cell and reproduces viral copies of it. Retroviruses are made of RNA.

What is the least virulent form of the HIV virus?

HIV-2

Anaphylaxis Medical Management

Immediate aggressive treatment is the goal. At the first sign, the prescribed dose of epinephrine 1 : 1000 is given subcutaneously for mild symptoms. It may be repeated at 5- to 20-minute intervals as prescribed by the health care provider. Epinephrine produces bronchodilation and vasoconstriction and inhibits the further release of chemical mediators of hypersensitivity reactions from mast cells. The effects of epinephrine take only a few minutes. Epinephrine 1 : 10,000 at 5- to 10-minute intervals may be administered intramuscularly or intravenously for a severe reaction as prescribed by the health care provider. Diphenhydramine 50 to 100 mg may be given intramuscularly or intravenously as indicated for allergic signs and symptoms. If moderate to severe signs and symptoms occur, IV therapy with volume expanders and vasopressor agents such as dopamine (Intropin) may be initiated to prevent vascular collapse, and the patient may be intubated to prevent airway obstruction. Oxygen may be administered by nonrebreather mask. Intubation or a tracheostomy may be required for oxygen delivery if progressive hypoxia exists. Place the patient in a recumbent position, elevate the legs, and keep the patient warm.

Review of Mechanisms of the Immune Response

In cellular immunity the antigen is processed by macrophages and recognized by T cells. T cells produce lymphokines, which further attract macrophages and neutrophils to the site for phagocytosis, or cytotoxic killer T cells can respond directly.

Diagnostic Tests for Allergies

Laboratory studies are usually not necessary unless allergic signs and symptoms are severe or last an extended amount of time. A complete blood count with differential to identify the type of white blood cells that are elevated, skin testing, total serum IgE levels, and a specific IgE level for a particular allergen may be ordered. The latter test is called a radioallergosorbent test (RAST).

Which factors have been linked to increased mortality and morbidity from HIV?

Lack of access to adequate care

Types of White Blood Cells and Their Involvement in HIV Disease-Natural killer (NK) cells

Large granular lymphocytes involved in cellmediated immune response Able to kill target cells because target cells are coated with antibody that binds to receptors on the surface of NK cells, allowing the NK cell to attach Kill target cells by releasing a substance that triggers lysis (breakdown of cell wall) of cell ROLE IN HIV Counts and structure of these cells remain normal in patients with HIV infection, but they are functionally defective.

retrovirus

Lentivirus that contains reverse transcriptase, which is essential for reverse transcription "the production of a deoxyribonucleic acid [DNA] molecule from a ribonucleic acid [RNA] model".

Innate (Natural) and Adaptive (Acquired) Immunity

Memory CHARACTERISTICS INNATE (NATURAL) None ADAPTIVE (ACQUIRED) Present

Types of White Blood Cells and Their Involvement in HIV Disease- NEUTOPHILS

Normally constitute 50%-75% of all circulating leukocytes and are capable of phagocytosis Important in the inflammatory response and the first line of defense against infection Short lifespan ROLE IN HIV DISEASE Neutropenia (WBC deficiency) commonly occurs in advanced HIV disease. Drug-induced neutropenia is common, especially with drugs used to treat P. jiroveci pneumonia, toxoplasmosis, CMV retinitis, or colitis and with NRTIs.

Pros and Cons of Highly Active Antiretroviral Therapy (ART)

PROs • Minimizes chance of emergence of resistant strain of virus • May play a role in the reduction of HIV transmission • Slows disease progression • Improves quality of life CONs • Can be toxic • Frequent side effects • Complexity of drug and dosing regimens • Difficulty with adherence to regimen (nonadherence can result in treatment failure) • High cost

Psychosocial Issues

People who have received a diagnosis of HIV infection deal with a complex set of psychosocial issues. Often, they are uncertain, fearful, depressed, and isolated. HIV infection can be treated, but it is incurable and contagious. Many infected people feel isolated and abandoned by friends and family because of the stigma associated with HIV. Like the health care costs associated with many chronic diseases, the expenses involved in treating HIV infection are daunting for many patients

Complementary and Alternative Therapies

People with HIV disease often use nontraditional or complementary therapies, such as massage, acupuncture or acupressure, and biofeedback. Some patients use nutritional supplements or herbal remedies with the hope of alleviating the side effects of the disease and the medications

Nursing Interventions for the Patient With HIV Infection or HIV Disease

Prevent Infection • Wash hands frequently, and administer skin lubricants to patient and caregiver to prevent skin breakdown. • Use a gentle liquid soap. • Provide for daily showering or basin bath; do not give patient tub bath if rashes are present; avoid extremely hot temperatures. • Use a separate washcloth for lesions. • For oral hygiene, provide soft toothbrushes; nonabrasive toothpaste; and mouth rinses with sodium bicarbonate, saline, or lemon and hydrogen peroxide before and after meals and at bedtime. • Use measures to prevent skin impairment, such as turning sheets, air mattresses. • Elevate and support areas of edema. • Observe biopsy sites and intravenous insertion sites daily for signs of infection. • Change dressings at least every other day; avoid plastic occlusive dressings. • Prevent exposure to sources of microbes, such as plants or uncooked fresh fruits and vegetables. • Carry out measures to prevent spread of infection: Use gloves for contact with bodily secretions, double plastic bags to dispose of bodily secretions, bleach and water (1 : 10) solution for cleaning contaminated areas. Modify Alterations in Body Temperature 4812 • Administer prescribed antibiotics, intravenous fluids, or antipyretics. • Encourage fluid intake of more than 2500 mL/day. • Maintain daily intake and output records. • Weigh patient daily. • Provide tepid sponge baths and linen changes as necessary. • Instruct patient in deep-breathing and coughing exercises to prevent atelectasis and additional fever. Promote Good Nutrition • Provide instruction for high-calorie, high-protein, high-potassium, lowresidue diet. • Encourage high-calorie, high-potassium snacks. • Suggest foods that are easy to swallow (gelatin, yogurt, puddings) when dysphagia is present. • Advise patient to avoid foods that are spicy or acidic, rare meats, and raw fruits and vegetables. • Provide oral care before and after meals patient eats. • Encourage patient to get out of bed and sit up for meals, if possible. • Avoid odors by aerating room. • Refer patient and caregiver for appropriate dietary consultations. • Educate patient and caregiver about food safety and food preparation. Promote Self-Care • Assess realistic functional ability. • Plan, supervise, and assist with activities of daily living as necessary. • Encourage patient to be as active and independent as possible. • Assist patient with range-of-motion exercise to prevent contractures. • Provide equipment such as assistive eating devices, walkers, and commodes to promote patient independence. • To prevent fatigue, pace activities and schedule rest periods. Provide Counseling 4813 • Assess and support patient's coping mechanisms. • Explore with patient and significant others normality of grief. • Assist patient and significant others in acknowledging and planning for anticipated losses. • Provide information as desired and necessary, depending on patient's ability to understand. • Suggest appropriate religious support. • Facilitate participation in support groups or individual counseling as appropriate. HIV, Human immunodeficiency virus.

Anaphylaxis Patient Teaching

Reassure the patient during procedures. teach the patient about avoidance of the allergen, advise him or her to wear or carry medical alert identification, and teach the patient to prepare and administer epinephrine subcutaneously. If signs and symptoms are not treat can lead to death fast.

plasmapheresis

Removal of plasma that contains components causing or thought to cause disease. Also called plasma exchange because when the plasma is removed, it is replaced by substitution fluids such as saline or albumin.

Innate (Natural) and Adaptive (Acquired) Immunity

Response mechanisms CHARACTERISTICS INNATE (NATURAL) Nonspecific: Mononuclear phagocytic system; inflammatory response ADAPTIVE (ACQUIRED) Specific immune response: Humoral immunity, cellular immunity

anaphylactic shock

Severe, life-threatening hypersensitivity reaction to a previously encountered antigen.

What is the most common mode of transmission of HIV?

Sexual transmission

simian immunodeficiency virus (SIV)

Similar to zoonotic. Was noted in primates and probably crossed into humans by way of the hunting and consumption of these animals in Africa.

HIV disease

Symptomatic human immunodeficiency virus HIV infection that is not severe enough for a diagnosis of acquired immunodeficiency syndrome AIDS; symptoms of HIV disease are persistent unexplained fever, night sweats, diarrhea, weight loss, and fatigue.

Remember the invader

The ability to remember antigens that invaded the body in the past allows a quicker response if subsequent invasion by the same antigen occurs.

wasting

The loss of lean body mass as a result of illness.

Latex Allergies

The more frequent and prolonged the exposure, the greater the likelihood of developing a latex allergy. In addition to gloves, latex-containing products used in health care may include blood pressure cuffs, stethoscopes, tourniquets, IV tubing, syringes, electrode pads, oxygen masks, tracheal tubes, colostomy and ileostomy pouches, urinary catheters, anesthetic masks, and adhesive tape.

side effects associated with plasmapheresis

The most common complications are hypotension and citrate toxicity. Hypotension is usually the result of vasovagal reaction or transient volume changes. Citrate is used as an anticoagulant and may cause hypocalcemia, which may manifest as headache, paresthesias, and dizziness.

Anaphylaxis

The most severe IgE-mediated allergic reaction is anaphylaxis, or systemic reaction to allergens. Anaphylaxis, or anaphylactic shock, is an acute and potentially fatal hypersensitivity (allergic) reaction to an allergen. Allergens that cause anaphylaxis include the following: • Venoms • Drugs, such as penicillin and aspirin • Contrast media dyes • Insect stings (such as bees and wasps) • Foods such as eggs, shellfish, and peanuts • Latex • Vaccines The hypersensitivity reaction results in a sudden severe vasodilation as a consequence of the release of certain chemical mediators from mast cells. Vasodilation causes an increase in capillary permeability, which causes fluid to seep into the interstitial space from the vascular space.

immunity

The quality of being unsusceptible to or unaffected by a particular disease condition.

Review of Mechanisms of the Immune Response

The skin and mucous membranes are natural barriers to infectious agents. When these barriers are crossed, the immune response is triggered in the immunocompetent host.

Which statement most accurately describes normal changes of aging of the immune system?

The thymus gland decreases in size and activity with age.

Latex allergy Prevention

Use latex precaution protocols (remove all latex gloves from area, remove all items containing latex, and latex allergy/ alert signs) for patients identified as having a positive latex allergy test or a history of signs and symptoms related to latex exposure. Many health care facilities have created latex-free product carts that can be used for patients with latex allergies.

Common Allergens Causing Anaphylaxis-DRUGS

Vaccines Allergen extracts Enzymes Penicillins Sulfonamides Cephalosporins Dextrans Hormones Contrast media Anesthetic agents

immunodeficient state exists

When the immune system does not protect the body adequately.

phagocyte

White blood cells that are able to surround, engulf, and digest microorganisms and cellular debris.

zoonotic disease

a disease communicable from animals to humans under natural conditions; also know as zoonosis. HIV is known as zoonotic, an organism that has been able to cross from an animal species to humans. other types are; Zoonotic influenza. Salmonellosis. West Nile virus. Plague. Rabies. Brucellosis. Lyme disease.

Latex allergy- type I Allergic reaction

a response to the natural rubber latex proteins and occurs within minutes of contact with the proteins. These types of allergic reactions can range from skin erythema, urticaria, rhinitis, conjunctivitis, or asthma to full-blown anaphylactic shock. Systemic reactions to latex may result from exposure to protein via various routes, including the skin, mucous membranes, inhalation, or blood.

Infection

a threat to the immunosuppressed patient. Meticulous aseptic technique is required when caring for these individuals. Prophylactic antibiotic therapy may be advisable, and good skin care is necessary. The frequency of bedside visits for staff and family should be limited. Do not allow people with infection near the patient.

HIV Nursing considerations

a. Maintain issues of confidentiality surrounding HIV and acquired immunodeficiency syndrome (AIDS) testing. b. Follow prescribed state regulations and protocols related to reporting positive test results.

CD4-to-CD8 ratio

a. Monitors progression of HIV b. Normal ratio is approximately 2:1.

CD4 + T-cell count

a. Monitors the progression of HIV b. As the disease progresses, usually the number of CD4 + T cells decreases, with a resultant decrease in immunity. c. The normal CD4 + T-cell count is between 500 cells/L and 1600 cells/L. d. In general, the immune system remains healthy with CD4 + Tcell counts higher than 500 cells/L. e. Immune system problems occur when the CD4 + T-cell count is between 200 cells/L and 499 cells/L. f. Severe immune system problems occur when the CD4 + T-cell count is lower than 200 cells/L.

Malnutrition

also alters cell-mediated immune responses. When protein is deficient over a prolonged period, the thymus gland atrophies and lymphoid tissue decreases. In addition, susceptibility to infections increases.

Immunodeficiency Disorders

an abnormal condition of the immune system in which cellular or humoral immunity is inadequate and resistance to infection is decreased. The first evidence of immunodeficiency is an increased susceptibility to infection. The problem can manifest as recurrent or chronic infection. Unusually severe infection with complications or incomplete clearing of an infection also may indicate an underlying immunodeficiency

Anaphylaxis Nursing Interventions

begin by assessing respiratory status, including dyspnea, wheezing, and decreased breath sounds. Also assess circulatory status, including dysrhythmias, tachycardia, and hypotension. Monitor vital signs continually. Other assessments include (1) intake and output (I&O); (2) mental status, including anxiety, malaise, confusion, and coma; (3) skin status, including erythema, urticaria, cyanosis, and pallor; and (4) gastrointestinal status, including nausea, vomiting, diarrhea, and incontinence.

Hypersensitivity Disorders

characterized by an excessive reaction to a particular stimulus. A hypersensitivity reaction is an inappropriate and excessive response of the immune system to a sensitizing antigen. Hypersensitivity disorders arise when harmless substances, such as pollens, danders, foods, and chemicals, are recognized as foreign

Radiation

destroys lymphocytes either directly or through depletion of stem cells. As the radiation dose is increased, more bone marrow atrophies, leading to severe pancytopenia and severe suppression of immune function.

Hypersensitivity Assessment

detailed history, including the following: 1. Onset, nature, and progression of signs and symptoms 2. Aggravating and alleviating factors 3. Frequency and duration of signs and symptoms

Tissue rejection

does not occur immediately after transplantation. It takes several days for vascularization to occur. Seven to 10 days after blood supply is adequately established, sensitized lymphocytes may appear in sufficient numbers for sloughing to occur at the site.

Autoimmune Disorders

entail the development of an immune response (autoantibodies or a cellular immune response) to one's own tissues; thus these disorders are failures of the tolerance to "self." Autoimmune disorders may be described as an immune attack on the self and result from the failure to distinguish "self" protein (self-antigens) from "foreign" protein.

hypofunctional immune system

exists in young children and older adults.

Hodgkin's lymphoma

greatly impairs the cell-mediated immune response, and patients may die of severe viral or fungal infections. Viruses, especially rubella, may cause immunodeficiency by direct cytotoxic damage to lymphoid cells. Systemic infections can place such a demand on the immune system that resistance to a secondary or subsequent infection is impaired

Plasmapheresis

has been used to treat autoimmune diseases such as systemic lupus erythematosus, glomerulonephritis, myasthenia gravis, thrombocytopenic purpura, rheumatoid arthritis, and Guillain-Barré syndrome. The rationale is to remove pathologic substances present in plasma. Many disorders for which plasmapheresis is being used are characterized by circulating autoantibodies (usually of the immunoglobulin G [IgG] class) and antigen-antibody complexes. Immunosuppressive therapy prevents recovery of IgG production, and plasmapheresis prevents antibody rebound (in which posttreatment autoimmune antibody concentrations exceed their pretreatment level) that often occurs after immunosuppressive therapy.

Knowledge of the function of the immune system

has enabled medical experts to find a way to control the rejection process.

secondary Immunodeficiency disorders

if the deficiency is caused by illnesses or treatment. Drug-induced immunosuppression is the most common type of secondary immunodeficiency disorder. Immunosuppressive therapy is prescribed for 4755 patients to treat a wide variety of chronic diseases, including inflammatory, allergic, hematologic, neoplastic, and autoimmune disorders. Immunosuppressive therapy also is used to prevent rejection of a transplanted organ. Some of these immunosuppressive drugs are cyclosporine, mycophenolate mofetil (CellCept), and azathioprine. Immunosuppression is also a serious side effect of cytotoxic drugs used in cancer chemotherapy. Generalized leukopenia often results, leading to a decreased humoral and cell-mediated response. Therefore secondary infections are common in immunosuppressed patients

primary Immunodeficiency disorders

if the immune cells are improperly developed or absent. The basic categories of primary immunodeficiency disorders include the following: • Phagocytic defects • B cell deficiency • T cell deficiency • Combined B cell and T cell deficiency Rare and often serious, whereas secondary disorders are more common and may be less severe.

Immunodeficiency disorders

include AIDS, as well as therapy-induced immunodeficiency such as chemotherapeutic agents given for cancer and immunosuppressant meds given to prevent a reaction to organ transplantation. In addition, some individuals are born with immune deficiency disorders, in which the function or development of immune components is impaired

Latex allergy risk factors

include all of the following except: - Persons with multiple surgeries - Atopy - Rubber industry workers -Health care workers - Persons with an allergy to pollen - Persons with an allergy to bananas, avocados, kiwi fruit, tomatoes, apples, chestnuts and melons.

Lymphocytes

include the T and B cells and the large, granular lymphocytes also known as natural killer "or NK" cells. Approximately 70% to 80% of the lymphocytes are T cell lymphocytes. When activated, T cells release compounds called lymphokines.

Patient problems for patients with hypersensitivity

include the following: 1. Potential for Injury, related to exposure to allergen 2. Inability to Tolerate Activity, related to malaise 3. Potential for Infection, related to inflammation of protective mucous membranes

Latex allergy risk factors

include; Long-term exposure to latex products (e.g., health care personnel, individuals who have had multiple surgeries, rubber industry workers). A history of hay fever, asthma, and allergies to certain foods previously listed.

Adaptive immunity

includes humoral and cell-mediated immunity. The adaptive immune system's specificity results from the production of antibodies in the cells. Antibodies develop naturally after infection or artificially after vaccinations

Viral culture

involves placing the infected client's blood cells in a culture medium and measuring the amount of reverse transcriptase activity over a specified period of time.

Stage 3 HIV

is when the patient develops acquired immunodeficiency syndrome (AIDS). This is the most severe stage of the infection. Patients develop opportunistic infections because their immune systems have been damaged by the HIV infection. Their viral loads are elevated and they are very contagious. Their CD4 count drops to less than 200 cells/mm. They may experience fever, chills, sweating, swollen lymph nodes, weight loss, and weakness

Antigenic determinants on the cells

lead to graft rejection via the immune process. Therefore antigenic determinants in recipient tissue and donor tissue are matched as closely as possible before transplantation.

Antinuclear antibody (ANA) determination

lear antibody (ANA) determination 1. The ANA determination refers to a blood test used in the differential diagnosis of rheumatic diseases and for the detection of antinucleoprotein factors and patterns associated with certain autoimmune diseases. 2. The ANA titer is positive in most individuals diagnosed with systemic lupus erythematosus (SLE); it may also be positive in individuals with systemic sclerosis (scleroderma) or rheumatoid arthritis.

Stress

may alter the immune response. This effect involves interrelationships between the nervous, endocrine, and immune systems.

Chills seen with mild or moderate transfusion reactions

may be caused by the rapid infusion of cold blood, or may be due to a reaction against monocytes, granulocytes, lymphocytes, or leukocytes present in the donated blood.

Viral load testing

measures the presence of HIV viral genetic material (RNA) or another viral protein in the client's blood.

severe transfusion reaction

often includes fever, chills, pain in the lower back, tightness in the chest, tachycardia, a drop in blood pressure, and blood in the urine. Patients may report a sense of "impending doom." Hemolytic reactions can result in kidney damage requiring dialysis, or death if the transfusion is not stopped as soon as the first signs of a reaction appear. If a severe reaction occurs, the transfusion is stopped immediately, saline is administered to maintain venous access, and the blood or blood product, along with the tubing, is returned to the laboratory for immediate testing. Further nursing responsibilities after a blood transfusion reaction include (1) following hospital protocol for collecting required blood and urine specimens to assess for hemolysis, (2) filling out a transfusion reaction record, (3) documenting the transfusion reaction on required facility forms and in the patient's health record.

T cells

originate in red bone marrow. Processed in the thymus. cooperate with the B cells to produce antibodies but do not produce antibodies themselves. responsible for cell-mediated immunity and protect the body against viruses, bacteria, fungi, and parasites. provide protection in allografts (transfer of tissue between two genetically dissimilar individuals of the same species) and against malignant cells.

severity of altered immune response disorders

ranges from mild to chronic to life threatening

Kaposi's sarcoma

rare cancer of the skin and mucous membranes characterized by blue, red, or purple raised lesions (Fig. 56.1) that occurred mainly in men of Mediterranean descent.

Anaphylaxis Clinical Manifestations

reaction occurs rapidly after exposure, from seconds to a few minutes. A massive release of mediators initiates events in target organs throughout the body. Skin and gastrointestinal signs and symptoms may occur, although respiratory and cardiovascular signs and symptoms predominate. Fatal reactions are associated with a fall in blood pressure, laryngeal edema, and bronchospasm, leading to cardiovascular collapse, myocardial infarction, and respiratory failure. Anaphylactic reactions are classified as mild, moderate, and severe.

Moderate transfusion reactions

resulting in fever, chills, urticaria, and wheezing, occur after the first 30 minutes of administration. In the event of a moderate reaction, -stop the transfusion, -continue with saline, and notify the health care provider. -Antihistamines and epinephrine may be ordered. -The health care provider will decide whether to continue the transfusion.

Patient Teaching

revolve around the specific diagnosis. Advise the patient with seasonal allergies to avoid offending allergens and ensure that he or she understands the therapeutic medication plan. Focus on health promotion and health teaching for self-care management.

Type I allergic reaction to latex is a response to the __________

rubber latex proteins

Anaphylaxis Assessment

the more rapid the onset, the more severe the outcome is. The patient may have a feeling of uneasiness that increases to a sense of severe apprehension, and then a fear of impending death. SKIN Urticaria, pruritus, and angioedema may be present in mild and moderate anaphylaxis, whereas cyanosis and pallor may be seen in severe reactions. Upper respiratory signs and symptoms range from congestion and sneezing to edema of the lips, the tongue, and the larynx with stridor and occlusion of the upper airways. Lower respiratory signs and symptoms occur soon thereafter and include bronchospasm, wheezing, and severe dyspnea. Gastrointestinal signs and symptoms increase from nausea, vomiting, and diarrhea to dysphagia and involuntary stools. The patient may have cardiovascular signs and symptoms, such as tachycardia and hypotension. Signs and symptoms may worsen, and the patient may display coronary insufficiency, vascular collapse, dysrhythmias, shock, cardiac arrest, respiratory failure, and death.

Plasmapheresis involves

the removal of whole blood through a needle inserted in one arm and circulation of the blood through a cell separator. The separator divides the blood into plasma and its cellular components by centrifugation or membrane filtration. Plasma, platelets, white blood cells, or red blood cells can be separated selectively. The undesirable component is removed, and the remainder is returned to the patient through a vein in the opposite arm. The plasma is generally replaced with normal saline, lactated Ringer's solution, fresh frozen plasma, plasma protein fractions, or albumin. When blood is removed manually, only 500 mL may be taken at one time

Isografting

the transfer of tissue between genetically identical individuals e.g., identical twins. Because few humans are born with an identical sibling, the allograft is the most common form of tissue transplant

Allografting

the transplantation of tissue between members of the same species. Allografts commonly are used after trauma, especially on full-thickness burns, and in reconstructive surgery.

Autografting

the transplantation of tissue from one site to another on the same individual and is generally successful because there is a reduced likelihood of rejection.

Surgery

urgical removal of lymph nodes, thymus, or spleen can suppress the immune response. Splenectomy in children is especially dangerous and may lead to septicemia from simple respiratory tract infections.

disruption of homeostasis

when immune response is too weak or too vigorous.

Signs and Symptoms of HIV Infection

• Abdominal pain • Chills and fever • Cough (dry or productive) • Diarrhea • Disorientation • Dyspnea • Fatigue • Headache • Lymphadenopathy (any disorder of the lymph nodes or lymph vessels) • Malaise • Muscle or joint pain • Night sweats • Oral lesions • Shortness of breath • Skin rash • Sore throat • Weight loss HIV, Human immunodeficiency virus.

Common Opportunistic Diseases Associated With HIV

• Candidiasis of bronchi, trachea, esophagus, or lungs • Invasive cervical cancer • Coccidioidomycosis • Cryptococcosis • Cryptosporidiosis, chronic intestinal (greater than 1 month's duration) • Cytomegalovirus (CMV) disease (particularly CMV retinitis) • Encephalopathy, HIV-related • Herpes simplex: chronic ulcer(s) (greater than 1 month's duration); or bronchitis, pneumonitis, or esophagitis • Histoplasmosis • Isosporiasis, chronic intestinal (greater than 1 month's duration) • Kaposi's sarcoma • Lymphoma, multiple forms • Mycobacterium avium complex • Tuberculosis • Pneumocystis carinii pneumonia • Pneumonia, recurrent • Progressive multifocal leukoencephalopathy • Salmonella septicemia, recurrent • Toxoplasmosis of brain • Wasting syndrome resulting from HIV HIV, Human immunodeficiency virus.

Barriers to Adherence

• Caregivers' incomplete knowledge of the demands of patients' lives • Prejudice • Insufficient appointment time • Lack of resources for patients who have doubts or questions • Negative stereotypes about doctors • Insufficient multidisciplinary communication

Psychological Crisis Intervals in the Course of HIV Disease

• Diagnosis of HIV infection • Viral load testing • Increases in viral load • Initiation of antiretroviral therapy • Signs of treatment failure • Adding prophylaxis therapies (e.g., for Pneumocystis jiroveci pneumonia) • Occurrence of opportunistic illnesses • Change in antiretroviral treatment regimen • Illness or death in support networks • New treatment advances HIV, Human immunodeficiency virus.

Key Points

• HIV, a retrovirus, is the agent that causes HIV disease and AIDS. • Education on prevention is the best method for preventing HIV disease and AIDS. • In the United States, HIV most commonly is spread by sexual activities and sharing injection equipment. • AIDS is the third and final stage of HIV infection. • When HIV enters the body, its primary target is the immune system. 4850 • HIV is transmitted by three major routes: (1) anal and vaginal intercourse, (2) injecting drugs with contaminated needles or works, and (3) from infected mother to child. • Blood, semen, vaginal secretions, rectal secretions, and breast milk are the body fluids that most readily transmit HIV. • Each patient's risks for HIV infection must be assessed, and those at risk must be counseled about testing, behaviors that put them at risk, and how to eliminate or reduce those risks. • A positive result of an HIV antibody test does not mean the patient has AIDS. • A multidisciplinary care approach in which the patient is a primary member of the team is the most appropriate method of caring for patients with HIV disease because of their complex needs. • As HIV infection progresses, the immune system loses its ability to fight infectious agents and cancer cells. • Patients at risk for HIV infection should be encouraged to learn their HIV status. • A person infected with HIV virus can transmit the virus, regardless of whether signs and symptoms are present. • Barriers to HIV prevention include denial, fear, misinformation, and cultural and community norms. • CD4 + counts are important markers of disease progression and the status of the immune system. • The stigma of HIV disease, because of its association with drug use, homosexuality, and sexual transmission, is a major concern. • The 1993 expanded case definition of AIDS includes all HIVinfected people who have CD4 + T lymphocyte counts of less than 200/mm3 ; includes all people who have one or more of more than 20 opportunistic infections regardless of CD4 count. 4851 • Measuring viral load in the blood helps assess effectiveness of therapy and possibly adherence. • Adherence to medications is essential.

Important characteristics of viral load monitoring include the following

• In all clinical stages of the illness, HIV virus detection techniques identify measurable amounts of viral RNA copies in the plasma of most HIV-infected individuals. • Viral load can provide significant information used to predict the course of disease progression, initiate ART, measure the degree of antiretroviral effect achieved, and note the failure of a drug regimen. • Plasma levels of HIV RNA fall dramatically after effective ART. • Detection of HIV RNA in plasma does not indicate whether any virus is present in lymphoid or other tissues.

CDC Recommendations to adults with no contraindications

• Influenza vaccine every year • Tetanus, diphtheria (Td) vaccine every 10 years; Tetanus, diphtheria, and pertussis (Tdap) once after age 19 • Varicella vaccine: two doses if there is no evidence of immunity to varicella • Human papilloma vaccination (HPV) for women 19 to 26 years of age; three doses if not previously vaccinated • Human papilloma vaccination (HPV) for men 19 to 21 years of age; three doses if not previously vaccinated. If having sex with men, vaccine should be given for men ages 19 to 26 years if not previously vaccinated. • Herpes zoster: two doses adults aged 50 and older • Measles, mumps, and rubella (MMR): two doses if there is no evidence of immunity • Pneumococcal 13 (PCV13): one dose • Pneumococcal 23 (PCV23): one to three doses depending on indication. One dose is required after the age of 65 • Hepatitis A: two to three doses if at risk for hepatitis A for persons 1 year and older • Hepatitis B—three doses if at risk for hepatitis B available for all age groups • Meningococcal—one or more doses if at risk for meningitis available beginning at age 11 to 12 years • Haemophilus influenza type B—to one to three doses if at risk for Haemophilus influenza type B available for children under the age of 5 years and for older children and adults who have qualifying medical conditions

Safety Alert Treating the Patient With a Hypersensitivity Reaction

• List all of the patient's allergies on the chart, the nursing care plan, and the medication record. • After an allergic disorder is diagnosed, therapeutic treatment is aimed at reducing exposure to the offending allergen; treating the symptoms; and, if necessary, desensitizing the person through immunotherapy. • All health care workers must be prepared for the rare but life threatening anaphylactic reaction, which requires immediate medical and nursing interventions. • Instruct the patient to wear a medical alert bracelet listing the particular drug allergy. • For a patient allergic to insect stings, commercial bee sting kits containing epinephrine and a tourniquet are available. Teach the patient to apply the tourniquet and self-inject the subcutaneous epinephrine. The patient should wear a medical-alert bracelet and carry a bee sting kit whenever he or she goes outdoors.

Latex allergy Prevention

• Non-latex gloves are recommended for tasks that are not likely to involve contacts with infectious materials such as blood (e.g., food preparation, routine housekeeping, and maintenance). • Workers at high risk of allergic reaction should be screened periodically to detect symptoms early and control or eliminate latex exposure. • Appropriate work practices should be followed. For example, workers should wash their hands with a mild soap and dry thoroughly after removing latex gloves. Areas contaminated with latex-containing dust should be identified and cleaned, and ventilation filters and vacuum bags used in those areas should be changed frequently. • Workers should be provided with education programs and training materials about latex allergy. • Workers showing symptoms of latex allergy should consult a doctor experienced in treating the problem, and workers with a known allergy should avoid latex exposures, wear a medical alert bracelet, and follow their doctor's advice for dealing with allergic reactions.

Assessing the Patient With Allergies

• Obtain a comprehensive history that covers family allergies, past and present allergies, and social and environmental factors, especially the physical environment. • Identify the allergens that may have triggered a reaction. • Determine the time of year that an allergic reaction occurs as a clue to a seasonal allergen. • Obtain information about any over-the-counter or prescription medications used to treat allergies. • In addition to identification of the allergen, find out about the clinical manifestations and course of allergic reaction. • Ask about pets, trees, and plants on the property; air pollutants; and floor coverings, houseplants, and cooling and heating systems in the home and workplace. • Have the patient keep a daily or weekly food diary with a description of any untoward reactions. • Screen for any reaction to medication. • Review questions about the patient's lifestyle and stress level in connection with allergic symptoms.

Lifespan Considerations: Older Adults- Immune Disorder

• Older adults are at increased risk for inflammation and infections resulting from changes in natural defense mechanisms. • Pathogens are able to enter through breaks in fragile, dry skin, increasing the risk of skin infections. * Decreased movement of respiratory secretions increases the risk of respiratory tract infections. • Decreased production of saliva and gastric secretions increases the risk of gastrointestinal infections. • Decreased tear production increases the risk of eye inflammation and infections. • Structural changes in the urinary system that lead to urinary retention or stasis increase the risk of urinary tract infection. • Signs and symptoms of infection tend to be more subtle than those in younger individuals. Because older adults have decreased body temperature, fever may be more difficult to detect. Changes in behavior such as lethargy, fatigue, disorientation, irritability, and loss of appetite may be early signs of infection. • Immune system functioning declines with advanced age. Research is continuing. • The older adult's immune system continues to produce antibodies; therefore immunization for diseases such as pneumonia and influenza is recommended. • Older adults who have chronic illnesses are generally at increased risk for infection. • Medications commonly taken by the elderly such as antidepressants, statin drugs for elevated cholesterol, proton pump inhibitors for heartburn, and steroids also can reduce the immune system response.

Considerations for ART include the following

• Previous experience with ART may affect the efficacy 4806 of a proposed therapy, because the virus may have become resistant to medications taken by the patient in the past (e.g., zidovudine, lamivudine). • Certain combinations of antiretroviral drugs may reverse the resistance to a single drug. Recycling drugs previously taken can sometimes improve viral suppression. Incorrect dosing (timing) or incorrect usage (missed doses) can cause drug resistance. • Drug incompatibilities, similar side effect profiles, and toxic effects must be considered in the choice of a regimen. • The patient's commitment to taking medication each day must be considered. Inadequate adherence can lead to drug resistance and, ultimately, to drug failure. This point must be stressed to the patient. Adherence is paramount for survival and success of treatment.

Key Points

• The two major forms of immunity are innate (natural) and acquired (adaptive). • T lymphocytes, B lymphocytes, and macrophages are the three major cell types active in acquired immunity. • B lymphocytes produce antibodies. T lymphocytes do not produce antibodies but assist the B cells. T lymphocytes release lymphokines. • Macrophages trap, process, and present antigen to T lymphocytes. • Autoimmune disorders are failures of the body's tolerance to "self." • Plasmapheresis is used to treat autoimmune diseases such as systemic lupus erythematosus, glomerulonephritis, myasthenia gravis, thrombocytopenic purpura, rheumatoid arthritis, and Guillain-Barré syndrome. • Infection is a primary threat to the immunosuppressed patient. Aseptic technique is required when caring for these patients. Good skin care is necessary. • Careful selection of blood donors and careful typing and crossmatching of blood are important to prevent transfusion reaction. • Early recognition of signs followed by early treatment may decrease the severity of an allergic reaction. • The five factors influencing the hypersensitivity response are host response to allergen, exposure amount, nature of the allergen, route of allergen entry, and repeated exposure. • Two types of latex allergies are type IV allergic contact dermatitis and type I allergic reactions. Type IV is caused by the chemicals used in the manufacturing of latex gloves, whereas the type I allergic reaction is a response to natural rubber latex proteins.


Kaugnay na mga set ng pag-aaral

Topic 10: Activity-Based Costing

View Set

Introduction to Business Process: Chapter 2 Quiz

View Set

Ap Gov Chapter 6 Judiciary Study Guide

View Set